Вы находитесь на странице: 1из 14

ECTest ID: 161269

TarGATE16

www.gateforum.com

General Aptitude
Q. No: 1-5 Carry One Mark Each
1.

2.

3.

4.

She unnecessarily picked up a quarrel with Mr Kumar and stormed out of the party.
(A) picked a quarrel

(B) picked on a quarrel

(C) did a quarrel

(D) made a quarrel

Lionesses with worn out or missing teeth are not ostracized from their pride, but live out their
old age supported by the hunting of younger females.
(A) humiliated

(B) eliminated

(C) banished

(D) tortured

The population of tigers in the National Park is increasing steadily, and this is a source of
encouragement to those who have worked so hard to fund the conservation effort.
(A) steadily, and this

(B) steadily: which

(C) steadily; this trend

(D) steadily, this increase

The value obtained if the sum of (11)2 and (4)3 is subtracted from the square of 27 is
________.

5.

Atul does half as much work as Anshu in 4

of the time. If together they take 16 days to


5
complete the work, how much time shall Anshu take to do it?
(A) 23

(B) 25

(C) 26

(D) 30

Q. No: 6-10 Carry Two Mark Each


6.

Virtually all health experts agree that second-hand smoke poses a serious health risk. After
the publication of yet another research paper explicating the link between exposure to secondhand smoke and a shorter life span, some members of the State House of Representatives
proposed a ban on smoking in most public places in an attempt to promote quality of life and
length of lifespan.
Which of the following, if true, provides the most support for the actions of the State
Representatives?
(A) The amount of damaging chemicals and fumes released into the air by cigarette smoke is
far less than the amount released from automobiles, especially from older models.
(B) Banning smoking in most public places will not considerably reduce the percent of the
population in the state in question that smokes.
ICPIntensive Classroom Program eGATE-Live Internet Based Classes DLP TarGATE-All India Test Series
Leaders in GATE Preparation 65+ Centers across India
All rights reserved by Gateforum Educational Services Pvt. Ltd. No part of this booklet may be reproduced or utilized in any form without the written permission.

ECTest ID: 161269

TarGATE16

www.gateforum.com

(C) The state whose legislators are proposing the tough smoking legislation has a relatively
high percent of its population that smoke.
(D) Another state that enacted a similar law a decade ago saw a statistically significant drop
in lung-cancer rates among non-smokers.
7.

In Raisin in the Sun, Lorraine Hansberry does not reject integration or the economic and
moral promise of the American dream; rather, she remains loyal to this dream while looking,
realistically, at its incomplete realization. Once we recognize this dual vision, we can accept
the plays ironic nuances as deliberate social commentaries by Hansberry rather than as the
unintentional irony that Bigsby attributes to the work. Indeed, a curiously persistent refusal
to credit Hansberry with a capacity for intentional irony has led some critics to interpret the
plays thematic conflicts as mere confusion, contradiction, or eclecticism. Isaacs, for
example, cannot easily reconcile Hansberrys intense concern for her race with her ideal of
human reconciliation. But the plays complex view of Black self-esteem and human solidarity
as compatible is no more contradictory than Du Boiss famous, well-considered ideal of
ethnic self-awareness coexisting with human unity, or Fanons emphasis on an ideal
internationalism that also accommodates national identities and roles.
Indicate the best answer: The authors primary purpose in the passage is to
(A) Explain some critics refusal to consider Raisin in the Sun a deliberately ironic play
(B) Suggest that ironic nuances ally Raisin in the Sun with Du Boiss and Fanons writings
(C) Analyze the fundamental dramatic conflicts in Raisin in the Sun
(D) Emphasize the inclusion of contradictory elements in Raisin in the Sun

8.

Alicia lives in a town whose streets are on a grid system, with all streets running east-west or
north-south without breaks. Her school located on a corner, lies three blocks south and three
blocks east of his home, also located on a corner. If Alicia is equally likely to choose any
possible path from home to school, and if she only walks south or east, what is the probability
that she will walk south for the first two blocks?
x
If y 3 and 2 is a prime integer greater than 2, which of the following must be true?
y
I. x = y II. y = 1 III. x and y are prime integers.
(A) None

10.

(B) I only

(C) II only

(D) III only

There are 4 letters and 4 corresponding envelopes. If we put the 4 letters into the envelopes at
random, what is the probability that only one letter was into the exact envelope?
(A) 1/8
(B) 1/6
(C) 1/3
(D) 1/2

Electronics and Communication Engineering


Q. No: 1 25 Carry One Mark Each
1.

If the two regression lines are 8x 10y 66 0 and 40x 18y 214 then mean of x and y
are respectively
ICPIntensive Classroom Program eGATE-Live Internet Based Classes DLP TarGATE-All India Test Series
Leaders in GATE Preparation 65+ Centers across India
All rights reserved by Gateforum Educational Services Pvt. Ltd. No part of this booklet may be reproduced or utilized in any form without the written permission.

TarGATE16

ECTest ID: 161269

(A) 13, 17

(B) 17, 13

(C) 18, 15

www.gateforum.com

(D) 15, 18

2.

Find 1st approximation to a real root of the equation x3 2x 5 0 by Regula falsi method
between 2 and 3.
(A) 2.0466
(B) 2.0588
(C) 2.0614
(D) 2.0831

3.

The directional derivative of f xy2 z at (1,-1, 1) in the direction of the vector a i j 2k


is
3
3
5
5
(A)
(B)
(C)
(D)
6
6
6
6

4.

Evaluate

5.

The initial value of the transfer function X(s)

6.

An RL circuit is driven by a voltage source for a long time. At t = 3, the source was
disconnected. At t 3 , the inductor behaves like

te3t sin tdt

(A) Voltage source


(C) Current source
7.

808
is _________.
s(s 2 2s 101)

(B) Capacitor
(D) Resistor

Assertion (A): In an inductor, voltage lags current by 90O


Reason (R): V jL I
(A) Both A & R are true and R is the correct explanation of A
(B) Both A & R are true and R is not the correct explanation of A
(C) A is true, R is false
(D) A is false but R is true

8.

In the figure shown below, the switch K is kept first at position 1 and steady state condition is
reached. At t = 0, the switch is moved to position 2. Find the current when t 0 .

K
1

10

100V

20

0.5H

ICPIntensive Classroom Program eGATE-Live Internet Based Classes DLP TarGATE-All India Test Series
Leaders in GATE Preparation 65+ Centers across India
All rights reserved by Gateforum Educational Services Pvt. Ltd. No part of this booklet may be reproduced or utilized in any form without the written permission.

TarGATE16

ECTest ID: 161269

(A) 0.33e40t A
9.

(B) 3.33e40t A

(C) 0.33e4t A

www.gateforum.com

(D) 0.33e4t A

The output voltage, V0 of the circuit shown below is ________V.(Assume diodes are ideal)

5V
2k

D1

1V

V0

2V

D2
3V

D3

10.

The emitter current in transistor Q1 for the circuit shown below is __________mA.
5V

2k

2k

Q1

2V

Q2

3k

10V

11.

For the circuit shown below the collector to emitter voltage of transistor T1 is _________V.
10

4k

1k

6k
T2

T1
4k

6k
1k

ICPIntensive Classroom Program eGATE-Live Internet Based Classes DLP TarGATE-All India Test Series
Leaders in GATE Preparation 65+ Centers across India
All rights reserved by Gateforum Educational Services Pvt. Ltd. No part of this booklet may be reproduced or utilized in any form without the written permission.

TarGATE16

ECTest ID: 161269

www.gateforum.com

12.

Minimum number of NAND gates required to implement half adder is ________.

13.

For successive approximation type ADC using clock of frequency 125 kHz, maximum time
required to convert analog input into 8-bit digital output is _______ s.

14.

The number of comparators in a 5 bit flash ADC is __________.

15.

At high electric field for values usually greater than 105 V/cm drift velocity is,
(A) Linear varies with field
(B) Inversely varies with square root of the field
(C) Inversely varies with field
(D) Constant

16.

The intrinsic carrier concentration of a semiconductor depends on


(A) Fermi energy level
(B) Donor energy level
(C) Forbidden energy band
(D) Acceptor energy level

17. The input to an LTI system is u t 3 . If the impulse response is as shown in figure below find
the output at t=1 and t=3 respectively

(A) 7, 4

5
3
(B) 0, 1

10
(C) 4, 1

(D) 0, 7

18. A finite length input sequence applied to an LT1 system and the corresponding output is shown in
the figure. Find the impulse response of the system.

x n 1,1
(A) 1, 1,0,0

19.

y n 1,0, 1,0,0

h n

(B) 1, 1

(C) 1,0,1

(D) 1,1,1,1

An open loop system which is stable can be modified into


(A) An unstable closed loop system
(B) Stable closed loop system
(C) A & B
(D) None of these

ICPIntensive Classroom Program eGATE-Live Internet Based Classes DLP TarGATE-All India Test Series
Leaders in GATE Preparation 65+ Centers across India
All rights reserved by Gateforum Educational Services Pvt. Ltd. No part of this booklet may be reproduced or utilized in any form without the written permission.

ECTest ID: 161269

20.

TarGATE16

www.gateforum.com

Which of the below transfer function cannot be realized using adder, substractor,
(A) G1

1
G2

(B)

G1
1 G1 H

(C)

G1
G2

(D) G1

1
G2

21.

Characteristic equation of a system is s6 2s5 3s4 5s2 6s 10 . The system is


(A) Stable
(B) unstable
(C) Marginally stable
(D) cant say

22.

he capacity of a binary symmetric channel with cross over probability of 0.4 is


_________bits/channel

0.6
0.4

23.

0.4

0.6
Which of the following is/are correct?
(a) H x H x y I x; y
(b) H y H y x I x; y
(c) H x, y H x y H y x I x ; y
(A) a, b, c are correct
(C) b, c are correct

24.

An EM wave is incident at air conductor interface. If impedance of the conductor is


c 120, the ratio of transmitted and incident electric field E t Ei is
(A) -1

25.

(B) a & b are correct


(D) None

(B) 1

(C) C 60

(D) 2C 120

A plane electromagnetic wave travelling along +z direction has its electric field given by
E 5cos(t z)a x 5sin t z a y

(A) Left circular polarized


(C) Left elliptical polarized

(B) right circular polarized


(D) right elliptical polarized.

Q. No. 26 55 Carry Two Marks Each


26.

The system of equations in two variables x1 x 2 3, 2.5x1 bx 2 7.5 will have infinite
number of solutions if and only if b is equal to _________.

ICPIntensive Classroom Program eGATE-Live Internet Based Classes DLP TarGATE-All India Test Series
Leaders in GATE Preparation 65+ Centers across India
All rights reserved by Gateforum Educational Services Pvt. Ltd. No part of this booklet may be reproduced or utilized in any form without the written permission.

ECTest ID: 161269

27.

TarGATE16

www.gateforum.com

Let P be the matrix with (1,1,1,3), (1, 2, 3, 4), (2,3,4,9) as the rows. Consider the two
statements.
(I) All the three rows are linearly independent
(II) Rank of P is 3

28.

(A) Both the statements are true

(B) Both the statements are false

(C) (I) is true but (II) is false

(D) (I) is false but (II) is true

The integral

x
1 i

y2 2ixy dz

(A) is independent of the path joining 0 and 1 + i


(B) Not independent of the path joining 0 and 1 + i
(C) Value of the integral is zero
(D) None of these

29. The general solution of the D.E (x3D3+3x2D2+xD+1)y = 0 (x>0) is

(A) y C1x x1 2 C2 cos


ln x C3 sin
ln x
2

ln x C3 sin
ln x
(B) y C1x 2 x1 2 C2 cos

ln x C3 sin
ln x
(C) y C1x 1 x1 2 C2 cos
2

ln x C3 sin
ln x
(D) y C1x 2 x 1 2 C2 cos
2

30.

Consider the function f (x) 1 2cosx 2sin 2 x in[0, ]. which of the following is/are
TRUE?
1. f(x) has absolute maximum at x
2. f(x) has local maximum at x / 3.
(A) 1 only (B) 2 only

(C) both 1 and 2

(D) None of these

ICPIntensive Classroom Program eGATE-Live Internet Based Classes DLP TarGATE-All India Test Series
Leaders in GATE Preparation 65+ Centers across India
All rights reserved by Gateforum Educational Services Pvt. Ltd. No part of this booklet may be reproduced or utilized in any form without the written permission.

TarGATE16

ECTest ID: 161269

www.gateforum.com

31. The current i(t) through the circuit is

it

1H

5u t

(A) 2.5cost
32.

33.

1F

(B) 5sint

(C) 5cost

For a series RLC circuit with C 10F, bandwidth is 10 kHz and the resonant frequency is
20kHz. the values of R and L are
(A) R 10k, L 6.33H

(B) R 10, L 6.33H

(C) R 100, L 10.12H

(D) R 0.1k, L 0.63mH

The ABCD parameters of the network shown in the following figure is

j5

E1

j5

34.

(D) 10sint

j2

E2

J10 25
4 J7.5

(A)

1.5 0.5J J5 1

4 J7.5 J10 25

(B)

1.5 0.5J J5 1

J1 2.5
2 J0.5

(C)

1.5 0.5J J5 1

J10 5
4 J7.5

(D)

0.5 0.5J J5 5

Match the following

IS

Amp

RS

If

RL

1 Trans resis tan ce amp

If

Feedback
Amp
ICPIntensive Classroom Program eGATE-Live Internet Based Classes DLP TarGATE-All India Test Series
Leaders in GATE Preparation 65+ Centers across India
All rights reserved by Gateforum Educational Services Pvt. Ltd. No part of this booklet may be reproduced or utilized in any form without the written permission.

TarGATE16

ECTest ID: 161269

www.gateforum.com

RS

VS

Vf

Amp

RL

Current amp

Transconduc tan ce amp

Feed
back
amp

IS

Amp

RS

C
If

RL

If

Feedback
Amp

35.

(A) A 1, B 2, C 3

(B) A 2, B 3, C 1

(C) A 3, B 2, C 1

(D) None of these

High cut-off frequency for the following second order low-pass filter is _________kHz.

5k
3k

25k

V0

30k
0.25nF

VS

0.5nF

ICPIntensive Classroom Program eGATE-Live Internet Based Classes DLP TarGATE-All India Test Series
Leaders in GATE Preparation 65+ Centers across India
All rights reserved by Gateforum Educational Services Pvt. Ltd. No part of this booklet may be reproduced or utilized in any form without the written permission.

TarGATE16

ECTest ID: 161269

36.

www.gateforum.com

For following voltage to current converter load current for voltage input of 7V is
___________mA.

2k

2k

V0

2k

2k

7V
IL

37.

Load

Match the following.


1-SAR ADC a-Fastest conversion
2-Flash type ADC b-Most accurate
3-Dual slope ADC c-Fixed conversion time
4-Voltage to frequency ADC d-Noisy industrial environment

38.

(A) 1 d 2 a 3 c 4 b

(B) 1 a 2 d 3 c 4 b

(C) 1 c 2 b 3 d 4 a

(D) 1 c 2 a 3 b 4 d

The truth table of a new XYFF is given below. To convert JKFF to XYFF, J and K,
respectively should be connected to
X Y Q n 1
0

Qn

Qn

(A) X, Y
39.

(B) X, Y

MVI

B, BC H

M0V

A, B

ADI

D2 H

(C) X Y,X Y

(D) XY,X

Status of flags S, Z, Carry and Parity after the execution of instruction ADI D2 H is
(A) 1,0,1,0

(B) 0,0,1,1

(C) 1,0,1,1

(D) 1,0,0,1

ICPIntensive Classroom Program eGATE-Live Internet Based Classes DLP TarGATE-All India Test Series
Leaders in GATE Preparation 65+ Centers across India
All rights reserved by Gateforum Educational Services Pvt. Ltd. No part of this booklet may be reproduced or utilized in any form without the written permission.

10

ECTest ID: 161269

40.

TarGATE16

www.gateforum.com

An abrupt silicon P+n junction has an n-region doping concentration of Nd 5 1015 cm3 .
The minimum n-region width such that avalanche break down occurs before the depletion
region reaches on ohmic contact is _________ m. Consider VB 100V
(Ignore the effect of built in potential and r(Si) 11.7 )

41.

An n-type Si-bar is 2cm long and has a cross-section of 2mm2mm. When a supply of 2V is
connected across it, 16mA current flows through it. If n 1300cm2 V s the doping level is
________ 1012 atoms / cm3 .

42.

In a BJT, the reverse saturation current Ico


(A) Doubles for every 10o rise in temperature
(B) Doubles for every 1o rise in temperature
(C) Doubles for every 5o rise in temperature
(D) Doubles for every 50o rise in temperature

43.

The input x[n] and output y[n] of a causal LTI system are related through the block-diagram
representation shown in fig.

x n

y n

z -1
1/ 2
Determine a difference equation relating y[n] and x[n]
1
1 z 1
2
(A)
1 z 1

44.

Assertion (A):

1
1 z 1
2
(B)
1 z 1

1 z 1
(C)
1
1 z 1
2

1 z 1
(D)
1
1 z 1
2

d 2 y(t) dy(t)

2y(t) = x(t) is stable when ROC is in between two poles


dt 2
dt

Reason (R): For any system to be stable, its ROC should lie between two poles
(A) Both A and R are true and R is the correct explanation of A
(B) Both A and R are true but R is not correct explanation of A
(C) A is true but R is false
ICPIntensive Classroom Program eGATE-Live Internet Based Classes DLP TarGATE-All India Test Series
Leaders in GATE Preparation 65+ Centers across India
All rights reserved by Gateforum Educational Services Pvt. Ltd. No part of this booklet may be reproduced or utilized in any form without the written permission.

11

TarGATE16

ECTest ID: 161269

www.gateforum.com

(D) A is false but R is true


45.

Consider the signal shown below.

x(t)

2
1

2
4 t in seconds
The value of x 2t 1 at t 0.5 is _________.

46.

For the signal flow graph shown in figure the value of

C S
R(S)

is

H3

R S

x3

x1
G1

x2

G2

C S

x4

G3

H1

G1 G 2 G3 G 4
(A)
1 G1 G 2 H1 G 3 G 4 G 2 G 3 H3 G1 G 2 G 3 G 4 H1 H 2

(B)

G1 G 2 G3G 4
1 G1 G 2 H1 G3 G 4 H 2 G 2 G 3 H3 G1 G 2 G 3 G 4 H1 H 2

(C)

1
1 G1G 2 H1 G3 G 4 H 2 G 2 G 3 H3 G1 G 2 G 3 G 4 H1 H 2

x5

G4

H 2

(D) None

47.

For the unity feedback system with forward path transfer function G S

K s 3 s 5

s 2 s 4

the range of K for the closed loop system to be stable is


(A) K>0.75or K<1
(B) K<0.75or K>-1
(C) K<0.75or K< -1
(D) K>0.75or K< -1

ICPIntensive Classroom Program eGATE-Live Internet Based Classes DLP TarGATE-All India Test Series
Leaders in GATE Preparation 65+ Centers across India
All rights reserved by Gateforum Educational Services Pvt. Ltd. No part of this booklet may be reproduced or utilized in any form without the written permission.

12

TarGATE16

ECTest ID: 161269

48.

www.gateforum.com

The time constant of a first order system is 2.5. Find the delay time (sec), if the maximum
value of output at steady state is unity.

49. Find the gain margin and phase margins of the feedback system with G S H S
(A) 0dB, 0O
50.

Consider a binary digital communication system which uses pulses g(t) & -g(t) for bandwidth
with over an AGWN channel. If the receiver uses a matched filter, which of the following
pulses will give minimum probability of error.

gt

B
g t 1

1
t

1.

C g t

D g t

1
1

51.

s 2

(D) , 0O

(C) 0dB,

(B) ,

Consider white Noise with two sided power spectral density of


shown Both LPF are identical

white noise

3e j2 fto

N0
passed by two systems as
2

Ideal LPF

PSD1

white noise

II

Ideal LPF

PSD2
The ratio of output noise power of system 1 to output noise power of system 2 is _________.

52.

A BPSK scheme is operating over an AWGN noise PSD of


S1

N0

uses equiprobable signal

2E
2E
sin c t , S2
sin t. If the local oscillator in a coherent receiver is ahead in
T
T

ICPIntensive Classroom Program eGATE-Live Internet Based Classes DLP TarGATE-All India Test Series
Leaders in GATE Preparation 65+ Centers across India
All rights reserved by Gateforum Educational Services Pvt. Ltd. No part of this booklet may be reproduced or utilized in any form without the written permission.

13

ECTest ID: 161269

TarGATE16

www.gateforum.com

phase by 30O with respect to the received signal, the probability of error in the resulting
system is
3E
(A) Q
2N
0

53.

54.

3E
(B) Q
N
0

2E
(C) Q
N
0

(D) None

After travelling ________ m in conductor with 5 10 s / m, r 3, r 4 wave with


frequency 5 MHz will get attenuated to 37% of its original value.
A plane wave propagating through a medium r 16, r 4, & 0 has its magnetic field
given by H= 5sin t x a z A / m . The wave impedance is _________ .

55.

A transmission line having 100 impedance is terminated in a load of 50 75j . The


VSWR is ________.

ICPIntensive Classroom Program eGATE-Live Internet Based Classes DLP TarGATE-All India Test Series
Leaders in GATE Preparation 65+ Centers across India
All rights reserved by Gateforum Educational Services Pvt. Ltd. No part of this booklet may be reproduced or utilized in any form without the written permission.

14

Вам также может понравиться